PT23.S2.Q14 - kim: in northern europe during the 18th century

Kimberly-207Kimberly-207 Member
edited January 2016 in Logical Reasoning 12 karma
http://7sage.com/lsat_explanations/lsat-23-section-2-question-14/
For a question stem like this are we strengthening Kim's argument or weakening Lee's? My initial thought was that it strengthens Kim's, but as it directly addresses something Lee says, I wanted to confirm.

Here's the stem: "Which one of the following, if true, provides the strongest defense of Kim's explanation against Lee's criticism?"

Comments

  • c.janson35c.janson35 Free Trial Inactive Sage Inactive ⭐
    edited July 2015 2398 karma
    This is definitely a weird question stem! My interpretation of it is that we are looking for an answer choice that would strengthen Kim's explanation by refuting Lee's criticism of her explanation. Lee definitely points out a potential weakness in Kim's argument, so the elimination of that weakness would strengthen Kim's argument in a sort of addition by subtraction kind of way.

    I think this should help lead you to the correct answer, but if you have any more questions feel free to ask!
  • NYC12345NYC12345 Alum Inactive Sage
    1654 karma
    It's a strengthening question.
Sign In or Register to comment.